People with higher-than-average blood levels of a normal dietary by-product called homocysteine are twice as likely t...

GLEE on December 24, 2018

Why E?

Could someone please explain why the answer is E? Also, why can't it be C? Thank you in advance.

Replies
Create a free account to read and take part in forum discussions.

Already have an account? log in

Ravi on December 24, 2018

@SROTD702,

Happy to help!

The issue in the stimulus deals with rent control, a policy designed to help protect tenants from steep increases in rents. The stimulus concludes that this practice can be detrimental, as it can create incentives for landlords to refrain from maintaining the quality of their existing properties and constructing new rental sites.

Answer choice A does nothing to explain the landlords' reluctance described in the stimulus. For one, A discusses non-rent-controlled housing, and this is not discussed in the stimulus, so it's irrelevant. It doesn't matter what the tenants prefer, as the fact of the matter is that the tenants discussed in the stimulus live in rent-controlled buildings, so the landlords presumably don't even have the option of providing more luxurious, non-rent-controlled units on these particular sites.

Answer choice B states that rent control makes it very difficult for landlords to achieve reasonable returns on any investments in maintenance or in new construction. This answer does a great job in explaining the landlords' reluctance described in the stimulus. Although rent control is intended to aid the renters, the rent controls can make it more challenging for landlords to earn returns on their investments in building or maintaining properties. This would explain their reluctance. If their profits are capped from rent control, why waste money on maintaining or building new units?

Does this make sense? Let us know if you have more questions!

Ravi on December 24, 2018

@SROT702 the above response in this thread; when copy-and-pasting my answer over from my word processor into this box, I mistakenly copied a different answer that I wrote, so I apologize.

@GLEE,

The stimulus gives us information that people who have higher-than-average levels of homocysteine are twice as likely to be diagnosed with Alzheimer's disease as are those with average or below-average homocysteine levels.

The stimulus then concludes that the risk of developing Alzheimer's disease could be reduced by including in one's diet large amounts of B vitamins and folic acid, which convert homocysteine into substances known to have no relation to Alzheimer's disease.

O.K., but so what? What if the people who have Alzheimer's disease are releasing more homocysteine into their blood than those who do not have Alzheimer's disease? The problem with this stimulus is that it assumes that if A (increased homocysteine levels) is correlated with B (higher likelihood of being diagnosed with Alzheimer's disease), then A must cause B.

This is mistaking correlation for causation, and it's just as possible that B is causing A, or some other variable is causing one or both of A and B. This is the flaw of the argument.

The question stem asks us to pick an answer that weakens the argument. Since we've identified the argument's flaw, we're looking for an answer that helps bring this flaw to light, exposing the argument and making it fall apart.

Answer A does not weaken the argument. So what if many Alzheimer's patients have normal homocysteine levels? This is consistent with the correlation that increased levels of homocysteine are associated with a higher likelihood of being diagnosed with Alzheimer's disease. Correlations are not conditional statements, so they allow for exceptions.

Answer B has nothing to do with the argument. The argument is solely focused on Alzheimer's disease; we don't care if the conversion of homocysteine into other substances can sometimes have harmful effects unrelated to Alzheimer's disease.

Answer C is irrelevant. We don't care about how efficiently B vitamins are metabolized by the body. If C were true, then people could just take vitamin B intravenously or through another method with more bioavailability. Vitamin-mineral supplements aren't the only way to ingest vitamin B.

Answer D suggests that there is a genetic component to Alzheimer's. However, it's still consistent with Alzheimer's having another cause, and it could also be consistent with the flawed argument in the stimulus. This answer does nothing to weaken the argument.

Answer E is exactly what we're looking for. Answer E tells us that B (Alzheimer's disease) actually causes A (increased levels of homocysteine). If this were true, it makes the stimulus's argument that A causes B fall apart. This points out the flaw that we identified earlier in our analysis of the argument, and it's our answer.

Does this make sense? Let us know if you have any more questions!

DevinFuller on July 27 at 08:07PM

So its a causal argument and us providing a different cause weakens the argument. Yes?

Emil-Kunkin on July 30 at 10:21PM

Yep! More specifically we're showing that causality might be the opposite direction, that there is an alternative cause from that the author says, the real chase being the thing that the author thought was the effect.